disuguaglianza istruttiva

Polinomi, disuguaglianze, numeri complessi, ...
Rispondi
Avatar utente
frengo
Messaggi: 223
Iscritto il: 01 gen 1970, 01:00

disuguaglianza istruttiva

Messaggio da frengo »

siano $ x,y,z $ numeri reali non negativi tali che $ x+y+z=1 $.

Dimostrare che

$ \displaystyle x^2y+y^2z+z^2x\leq\frac{4}{27} $

ciao ciao

ps perchè "istruttiva" lo spiegherò in seguito...
Avatar utente
HumanTorch
Messaggi: 281
Iscritto il: 01 gen 1970, 01:00
Località: Tricase

Messaggio da HumanTorch »

stanco stanco, quindi posto solo qualche approccio, poi domani vedo se s'è accordo 8) :

.se $ xyz\neq 0 $ dividiamo tutto per xyz: da $ \displaystyle\frac{\frac{x}{z}+\frac{y}{x}+\frac{z}{y}}{3}\leq \frac{4}{81xyz} $.

essendo $ a=\frac{x}{z} $ et similia, abc=1

poi sul LHS andiamo sul riordinamento e sulle disuguaglianze fra medie

sottraiamo 3xyz a entrambi i membri e scomponiamo, poi di nuovo medie:

oppure...non so, assegnare a $ x:=\tan\alpha\cdot \tan\beta $, $ y:=\tan\alpha\cdot \tan\gamma $, $ z:=\tan\gamma\cdot \tan\beta $ con $ \alpha +\beta+\gamma=\pi $

o altre scomposizioni, o magari moltiplicando LHS o RHS per 1=x+y+z e rendendo l'equazione omogenea?

Fate vobis, ora sphackha mi aspetta 8)
Avatar utente
frengo
Messaggi: 223
Iscritto il: 01 gen 1970, 01:00

Messaggio da frengo »

ehm...potresti essere più preciso?

il vero problema di questa disuguaglianza è che le medie e le altre disuguaglianze classiche NON SI POSSONO USARE(o al massimo in maniera MOLTO MOLTO furba)perchè l'uguaglianza non vale quando $ x=y=z $ come nella gran parte delle disuguaglianze.
aspetto chiarimenti

ciao ciao

ps hint:quando vale l'uguaglianza?
darkcrystal
Messaggi: 706
Iscritto il: 14 set 2005, 11:39
Località: Chiavari

Messaggio da darkcrystal »

Non sono sicuro al 100% di questa soluzione, provo lo stesso...
supponiamo wlog $ x \geq y \geq z $. Voglio provare che vale
$ f(x+z,y,0) \geq f(x,y,z) $ per poter poi porre z=0
Devo quindi provare $ x^2y+z^2y+2xyz \geq x^2y+y^2z+z^2x $
$ z^2y + yz(x-y)+xz(y-z)>=0 $ che è sicuramente vera perchè è una somma di quantità positive.
Perciò possiamo "stringere" la disuguaglianza dicendo che vogliamo provare $ \displaystyle f(x,y,0) \leq \frac{4}{27} $. Ma questa è facile: $ \displaystyle x^2(1-x) \leq \frac{4}{27} $ si può fare ad esempio vedendo dove si annulla la derivata prima, e scoprendo così che l'eguaglianza si ha per $ (\frac23,\frac13,0) $
Spero sia giusta, ciao!
"Solo due cose sono infinite: l'universo e la stupidità dell'uomo, e non sono tanto sicuro della prima" - Einstein

Membro dell'EATO
Avatar utente
HumanTorch
Messaggi: 281
Iscritto il: 01 gen 1970, 01:00
Località: Tricase

Messaggio da HumanTorch »

mmmh, si, contavo di trovare qualcosa annullando una delle variabili, ma non ho ricavato niente. complimenti a Darkcrystal per l'elegantissima soluzione 8) :D
Avatar utente
frengo
Messaggi: 223
Iscritto il: 01 gen 1970, 01:00

Messaggio da frengo »

no, manca qualcosa...visto che la disuguaglianza non è simmetrica, se hai $ f(x;y;z) $ non puoi supporre $ x\geq y\geq z $.Essendo però ciclica invece che tutti e sei i casi te ne bastano due, ma credo che anche il secondo caso si possa dimostrare in modo analogo...

ciao ciao

ps oggi pomeriggio posto la mia soluzione
darkcrystal
Messaggi: 706
Iscritto il: 14 set 2005, 11:39
Località: Chiavari

Messaggio da darkcrystal »

C'era qualcosa che non mi tornava, infatti... grazie della spiegazione!
"Solo due cose sono infinite: l'universo e la stupidità dell'uomo, e non sono tanto sicuro della prima" - Einstein

Membro dell'EATO
Avatar utente
frengo
Messaggi: 223
Iscritto il: 01 gen 1970, 01:00

Re: disuguaglianza istruttiva

Messaggio da frengo »

ecco la mia soluzione, è "istruttiva" perchè usa più o meno nozioni che si apprendono in una scuola superiore

$ \displaystyle x^2y+y^2z+z^2x\leq\frac{4}{27} $

visto che la condizione è $ x+y+z=1 $ sostituisco $ z=1-x-y $ con $ z\geq\frac{1}{3} $ (una delle tre incognite lo deve essere per forza); x e y diventano così due numeri variabili tra 0 e 2/3(ma la cui somma dev'essere anch'essa minore di 2/3).

$ \displaystyle x^2y+y^2(1-x-y)+(1-x-y)^2x\leq\frac{4}{27} $

$ \displaystyle x^2y+y^2-xy^2-y^3+x+x^3+xy^2+2x^2y-2xy-2x^2\leq\frac{4}{27} $

$ \displaystyle -y^3+y^2+(3x^2-2x)y+x(1-x)^2\leq\frac{4}{27} $

vista in y, il lato sinistro dell a disuguaglianza può essere visto come un fascio di cubiche di cui dobbiamo trovare il massimo per x,y nell'intervallo.vediamo dove sono i punti estremanti di queste funzioni:

$ f_x(y)=-y^3+y^2+(3x^2-2x)y+x(1-x)^2 $

$ f'_x(y)=-3y^2+2y+3x^2-2x $

risolvendo l'equazione f'_x(y)=0

$ -3y^2+2y+3x^2-2x=0 $

$ (x-y)(3x+3y-2)=0 $

$ y=x $
$ \displaystyle y=\frac{2}{3}-x $

notiamo che tutti e due i punti estremanti sono per $ 0\leq y\leq\frac{2}{3} $.

visto che la funzione è una cubica, non è complicato prevederne l'andamento:la funzione:
-decresce da $ 0 $ a $ \min\{x;\frac{2}{3}-x\} $
-cresce da $ \min\{x;\frac{2}{3}-x\} $ a $ \max\{x;\frac{2}{3}-x\} $
-decresce da $ \max\{x;\frac{2}{3}-x\} $ a $ \frac{2}{3} $.

i punti quindi in cui la funzione può avere il massimo sono 0 e $ \max\{x;\frac{2}{3}-x\} $.

per avere il massimo, quindi, y può valere solo 0,x e 2/3-x.sostituiamo questi valori nella funzione:

y=0

$ \displaystyle f_x(0)=x(1-x)^2=\frac{2x\cdot (1-x)\cdot (1-x)}{2} $$ \displaystyle \leq\frac{1}{2}\left(\frac{2x+1-x+1-x}{3}\right)^3=\frac{4}{27} $

per AM-GM

y=x

$ \displaystyle f_x(x)=3x^3-3x^2+x=\frac{(3x-1)^3+1}{9} $

che è chiaramente una funzione in x monotona crescente e assume quindi il massimo per x più gtande possibile, quindi x=1/3 (perchè anche y=x quindi x+y=2x non può essere più grande di 2/3), in cui vale \frac{1}{9}<\frac{4}{27}.

y=2/3-x

$ \displaystyle f_x(\frac{2}{3}-x)=-x^3+x^2-\frac{1}{3}x+\frac{4}{27}=(\frac{1}{3}-x)^3+\frac{1}{9} $

che è chiaramente una funzione monotona decrescente in x e assume quindi il valore massimo in x=0, in cui vale$ \frac{4}{27}. $

avendo esaminato tutti i casi, posso dire che la funzione non diventa mai più grande di 4/27, e ho dimostrato la disuguaglianza.


secondo me questa soluzione è "istruttiva" perchè è meccanica e non richiede lampi di genio di alcun genere.si può usare abbastanza spesso, quando si hanno cose riconducibili a funzioni note.

dò altri problemi che si possono risolvere in questo modo:

1) siano x,y,z numeri reali compresi tra 0 e 1.

dimostrare che

$ x+y+z\leq 2+xyz $

2)il problema postato da Boll non lontano da qui

3) siano $ x_1,x_2...x_n $ numeri reali compresi tra 0 e 1.

dimostrare che

$ x_1+x_2+x_3...x_n-x_1x_2-x_2x_3...-x_nx_1\leq\left[\frac{n}{2}\right] $

4) siano x,y,z numeri reali compresi tra 0 e 1

trovare il massimo dell'espressione

$ (x-y)^4+(y-z)^4+(z-x)^4 $

5) siano x,y,z numeri reali tali che x+y+z=1

dimostrare che

$ 0\leq xy+yz+zx-2xyz\leq\frac{7}{27} $.

ciaociao

buona pasqua a tutti

Francesco
darkcrystal
Messaggi: 706
Iscritto il: 14 set 2005, 11:39
Località: Chiavari

Messaggio da darkcrystal »

Satollo di cioccolata, tento di smaltire con un po' di attività mentale...
Allora, per la nuova 1:
$ x-xyz+y+z \leq 2 $
$ x(1-yz)+y+z \leq 2 $
LHS è, rispetto ad x (ma si può fare nello stesso modo per gli altri...), una retta con coefficiente angolare positivo. Il suo massimo è quindi in x=1, e si ha:
$ 1-yz+y+z \leq 2 $
$ y+z-yz-1 \leq 0 $
$ (y-1)(1-z) \leq 0 $ che è vera perchè LHS è il prodotto di un positivo e un negativo per ipotesi. Facendolo sulle tre variabili si ottiene la tesi, e che per l'uguaglianza bastano due variabili uguali ad uno, e la terza libera.

Stavolta è ok, o il cioccolato mi ha dato alla testa?

Ciao a tutti!
"Solo due cose sono infinite: l'universo e la stupidità dell'uomo, e non sono tanto sicuro della prima" - Einstein

Membro dell'EATO
Avatar utente
enomis_costa88
Messaggi: 537
Iscritto il: 01 gen 1970, 01:00
Località: Brescia

Messaggio da enomis_costa88 »

Provo la prima proposta:

Ok non sono metodi scolastici ma sempre analisi è..

WLOG sia z minore sia di x che di y.
$ f(x)=x^2y+y^2z+z^2x $
$ g(x)=x+y+z=1 $
Usando i moltiplicatori di lagrange applicati alle funzioni f(x) e g(x):
Gli "interior extremum" sono in corrispondenza di:
$ \lambda=2xy+z^2=2yz+x^2=2xz+y^2 $
ovvero:
$ (x-y)(2z-x-y)=0 $
$ (z-x)(2y-z-x)=0 $
$ (z-y)(2x-y-z)=0 $
da cui ottengo facilmente $ x=y=z=\frac{1}{3} $ e $ f(x)= \frac{3}{27} $.

Devo ora analizzare le situazioni di bordo:
la funzione è definita sul piano $ x+y+z=1 $ con $ x,y,z\ge 0 $.
I bordi sono in corrispondenza del valore nullo di una variabile (WLOG z).
(x,y,0) = (x,1-x,0) mi da
$ x^2y = f(x) $ con
$ x+y=1 $
Uso i moltiplicatori di lagrange per cercare eventuali punti di massimo interni al bordo e ottengo:
$ \lambda=2xy=x^2 $ e poichè $ x\not = 0 $
$ 2y=x $ ovvero
$ (\frac{2}{3};\frac{1}{3},0) $
in corrispondenza del quale f(x)= $ \frac{4}{27} $.
Avendo cercato solo i massimi interni ai bordi devo ora valutare gli estremi dei bordi stessi ovvero(1,0,0) e (0,1,0) e (0,0,1) in cui f(x,y,z)=0

Buona serata, Simone.
Avatar utente
enomis_costa88
Messaggi: 537
Iscritto il: 01 gen 1970, 01:00
Località: Brescia

Messaggio da enomis_costa88 »

La prima delle nuove proposte:

Sia: f(x,y,z)=x+y+z-xyz
Posso poi imporre il vincolo
g(x,y,z)=x+y+z=k
e usare i moltiplicatori di Lagrange:
$ \lambda=1-yz=1-xy=1-zx $ da cui
1) y(z-x)=0
2) x(z-y)=0
3) z(x-y)=0
WLOG suppongo :
4) $ x \ge y \ge z $
dalla 1 ho che o x=z e quindi per la 4 x=y=z
oppure y=0 e per la 4 y=z=0
per cui gli "interior extremum" candidati punti di massimo sono (x,x,x) e (x,0,0)

Nel primo caso ottengo: f(x,x,x)=$ 3x-x^3 \leq 2 $ che si verifica per esempio come segue:
$ x-1 \leq 0 $
$ x^2+x-2 \leq 0 $ per cui
$ (x-1)(x^2+x-2)=x^3-3x+2 \ge 0 $

Nel secondo: f(x,0,0)= $ x\leq 1 < 2 $

Devo ora analizzare i valori assunti da f(x) in corrispondenza dei bordi del cubo in cui è definita.
A meno di simmetria posso analizzare solo le situazioni (1,y,z) e (x,y,0)

Nel secondo caso:
$ f(x,y,0)=x+y\leq 2 $

Nel primo caso:
$ f(1,y,z)=1+z+y-yz $ e impongo y+z=j
Trovo i candidati massimo interni al quadrato definito da x=1 e $ 0\leq y,z \leq 1 $ :
Usando i moltiplicatori di lagrange:
$ \lambda = 1-y=1-z $ ovvero y=z e
f(1,y,y)= $ 1+2y-y^2\leq 2 $ che si verifica facilmente.

Analizzo le situazioni di bordo del quadrato definito da x=1 e $ 0\leq y,z \leq 1 $
Che sono a meno di simmetria: (1,1,z) e (1,y,0)
In corrispondenza dei quali ho:
$ f(1,1,z)=2+z-z=2 $
e $ f(1,y,0)=1+z\leq 2 $
e ciò conclude la dimostrazione.

EDIT:corretto un passaggio.
Ultima modifica di enomis_costa88 il 18 apr 2006, 09:31, modificato 1 volta in totale.
Avatar utente
enomis_costa88
Messaggi: 537
Iscritto il: 01 gen 1970, 01:00
Località: Brescia

Messaggio da enomis_costa88 »

Provo la quinta:

sia $ f(a,b,c)=Q-2P $
$ g(a,b,c)=S=1 $
Usando i moltiplicatori di lagrange:
$ \lambda=a+b-2ab=a+c-2ac=b+c-2bc $
1)(b-c)(1-2a)=0
2)(c-a)(1-2b)=0
3)(a-b)(1-2c)=0
WLOG
4) $ a\ge b\ge c $
dalla 1 ottengo o che b=c e quindi per la due b=c=a (poichè per la 4 non è possibile $ b=c=\frac{1}{2} $ ).
Oppure $ a=\frac{1}{2} $ e per la 2 $ a=b=\frac{1}{2} $ non essendo possibile $ a=c = \frac{1}{2} $ per la 4
per cui gli "interior extremum" sono $ (\frac{1}{3};\frac{1}{3};\frac{1}{3}) $ e $ (\frac{1}{2};\frac{1}{2};0) $
nel primo caso ottengo $ f(a,b,c) = \frac{7}{27} $
nel secondo $ f(a,b,c)=\frac{1}{4} $

Devo ora analizzare le situazioni di bordo:
la funzione è definita sul piano a+b+c=1 con $ a,b,c\ge 0 $.
I bordi sono in corrispondenza del valore nullo di una variabile.
WLOG sia c=0.
Devo considerare $ f(a,1-a,0)= a(1-a) $
Per l’AM-GM:
$ \sqrt{a(1-a)}\leq \frac{1-a+a}{2}=\frac{1}{2} $ ed elevando ottengo:
$ f(a,1-a,0)= a(1-a)\leq \frac{1}{4} $
che conclude la prima parte della dimostrazione.

Seconda parte ovvero: $ f(a,b,c)\ge 0 $
$ Q-2P=QS-2P $ = $ 3abc+a^2b+ab^2+a^2c+ac^2+b^2c+bc^2-2abc $= $ abc+a^2b+ab^2+a^2c+ac^2+b^2c+bc^2\ge 0 $

ok ragazzi adesso la smetto :P

Buona serata, Simone.
EvaristeG
Site Admin
Messaggi: 4896
Iscritto il: 01 gen 1970, 01:00
Località: Roma
Contatta:

Messaggio da EvaristeG »

Ehm ... due cose :
[quella seria] non è bello postare nello stesso thread più esercizi, soprattutto non nel modo sopra fatto, ovvero in coda ad un messaggio di soluzione ... esercizi che non sono legati al primo se non dal tipo... quindi, per favore, le prossime volte fate attenzione.

[quella poco seria] "interior extrema" al plurale, "interior extremum" al singolare ... è un prestito dal latino ...
Rispondi